AAPMR QBank - Patient Evaluation and Diagnosis

¡Supera tus tareas y exámenes ahora con Quizwiz!

A 75-year-old man is found confused and wandering in the street at night wearing his night clothes. In the emergency department (ED), he appears disorderly and disheveled. He is alert, but disoriented in time and place and cannot recall his home address. He engages well with questions, but tends to shift the conversation to stories about his wife and children. He is admitted to the hospital for further care, but wanders around the ward appearing lost. When asked where he is going, he tells nursing personnel that he is looking for a bus stop to go home. The patient has: A.Alzheimer's Dementia B.Amnestic Mild Cognitive Impairment (MCI) C.Subdural Hematoma

A. Alzheimer's Dementia 86.55% n Alzheimer's Dementia (AD), memory impairment occurs first, followed by decline in language and visuospatial skills relatively early. In Amnestic MCI, an early stage of AD, there is limited anterograde long-term memory impairment, with preserved function, but this patient's disheveled appearance suggests functional decline. AD is characterized by: Memory impairment noted in learning or recall Aphasia, Apraxia, Agnosia or Dysexecutive function (planning, organizing, sequencing, abstracting) Cognitive deficits of sufficient severity to affect social or occupational functioning, representing a change from previous level The clinical course of AD is gradual onset and progression, with no delirium precipitants contributing to the clinical picture and no alternative central nervous system explanation (e.g., stroke, Parkinson's disease). There is no history provided in the question stem, such as a fall, to suggest brain injury.

Radiographic findings associated with ankylosing spondylitis include symmetric SI joint narrowing, osteopenia, and a "Bamboo spine." A "Bamboo spine" refers to ossification of which structure? A.Annulus fibrosis B.Ligamentum flavum C.Supraspinous ligament D.Interspinous ligament

A. Annulus fibrosis 29.13%This patient has ankylosing spondylitis. The hallmark is bilateral sacroiliitis. The formation of a bamboo spine on radiographic studies is secondary to ossification of the annulus fibrosis leading to bridging syndesmophytes. Interspinous ligament ossification can result in a "dagger sign" on xray.

Which of the following anatomical locations is used to assess C6 level sensation? A.At the dorsal surface of the proximal phalanx of the thumb B.Over the medial epicondyle just distal to the elbow C.At the dorsal aspect of the base of the middle finger D.Over the lateral epicondyle at the elbow

A. At the dorsal surface of the proximal phalanx of the thumb 88.97% This is a tested area for C6 sensation under the ASIA exam criteria. It correlates with wrist extensors, which are the tested muscle for determining C6 motor impairment.

A 33-year-old male T8 AIS A paraplegic complains of new headache. As the responding physician you notice piloerection, facial flushing, bradycardia, and systolic blood pressure of greater than 40 mmHg above baseline. What is the most likely diagnosis and pathophysiology affecting this patient? A.Autonomic dysreflexia due to sympathetic hyperactivity B.Primary hypertension C.Reduced cardiovascular tolerance due to reduced sympathetic efferent output D.Adrenal urgency due to excess sympathetic stimulation

A. Autonomic dysreflexia due to sympathetic hyperactivity 93.92% This patient is experiencing autonomic dysreflexia, which is defined as an increase of 40 mmHg systolic or 20 mmHg diastolic blood pressures. It is associated with spinal cord injuries above the T6 level but has been described in spinal cord injuries as low as T10. It is related to exaggerated response of the autonomic nervous system to an irritant occurring below the level of injury. It classically is associated with a headache, piloerection, facial flushing, mydriasis, and bradycardia. It is most commonly caused by bladder distention with the second most common cause being bowel distention. It can be caused by any irritant such as ingrown toenails, wounds or fractures. Treatment is based upon blood pressure control and removing the inciting factor concurrently.

A 62-year-old patient with long-standing history of diabetes mellitus presents to your clinic with insidious onset of numbness and tingling in the feet, orthostatic hypotension, chronic constipation, and urinary retention with overflow incontinence over the last several months. What is the most likely diagnosis? A.Autonomic neuropathy B.Diabetic amyotrophy C.Plexoradiculopathy D.Guillain-Barre Syndrome

A. Autonomic neuropathy 78.12% Long-standing diabetes mellitus can cause several different types of neuropathy. The addition of orthostatic hypotension, constipation or gastroparesis, urinary retention, erectile dysfunction, and abnormalities in thermoregulation suggest involvement of the autonomic nervous system. Diabetic amyotrophy is a proximal lumbosacral plexopathy causing severe pain, proximal weakness and muscle atrophy. Plexoradiculopathy can occur from many different causes, including uncontrolled diabetes and radiation therapy, and involves the proximal nerve roots and the plexus of the involved limb. Guillain-Barre Syndrome, or acute inflammatory demyelinating polyneuropathy, does cause ascending distal numbness and weakness as well as autonomic symptoms, but the onset is typically very rapid over several days to weeks.

A 3-year-old male presents with toe walking and elevated transaminases. What additional physical exam finding is most likely? A.Calf pseudohypertrophy B.Pes cavus C.Stereotyped movements D.Ankle plantarflexor spasticity

A. Calf pseudohypertrophy 79.33%Elevated transaminases are frequently found incidentally in boys with Duchenne muscular dystrophy (DMD). This is due to muscle breakdown as indicated by a significantly elevated CK level. Exam is notable for calf pseudohypertrophy. Stork-like appearance of legs and high arches are consistent with Charcot-Marie-Tooth disease. Stereotyped movements can be seen in autism spectrum disorders, which can also lead to toe walking due to sensory dysregulation. Hypertonicity and involuntary movements are features of cerebral palsy, which generally does not present with laboratory abnormalities.

A diabetic patient's foot examination is significant for a "rocker-bottom" deformity. X-rays of the patient's foot reveal extensive bone resorption, shattering, and sclerosis of the metatarsophalangeal joints and mid foot. Inflammatory markers are normal. The most likely diagnosis is: A.Charcot arthropathy B.Osteomyelitis C.Lisfranc fracture D.Osteoarthritis

A. Charcot arthropathy 97.09% Differentiating between osteomyelitis (OM) and an acute neuropathic arthropathy (ie, Charcot arthropathy) in the diabetic foot is challenging. The "rocker-bottom" deformity and disorganized features on x-ray of bone destruction and repair clinch the diagnosis. Prominent fragmentation and loose-body formation typically occurs in the tarsal bones, whereas resorptive changes may predominate in the metatarsals and forefoot. Bone scintigraphy with technetium 99m-MDP and indium 111-labeled WBC scans can be useful in differentiating between OM and neuropathic arthropathy. Preliminary evidence supports the use of nuclear medicine techniques to differentiate the marrow uptake seen in neuropathic arthropathy from the bone uptake seen with OM. The clinical history does not support the diagnosis of a Lisfranc fracture. The radiographic changes are inconsistent with osteoarthritis of the mid-foot.

Electrodiagnostic findings of marked slowing in nerve conduction velocity (less than 25 meters/second) without conduction block or temporal dispersion would be most consistent with: A.Charcot-Marie-Tooth disease, type 1 B.Myasthenic syndrome C.Guillain-Barré syndrome D.Amyloidosis

A. Charcot-Marie-Tooth disease, type 1 70.21% Hereditary motor sensory neuropathies (Charcot-Marie-Tooth disease, type 1) usually have conduction slowing without temporal dispersion of compound muscle action potentials. Acquired demyelinating conditions such as Guillain-Barré Syndrome or acute inflammatory demyelinating polyradiculoneuropathy (AIDP) usually have conduction block and temporal dispersion on nerve conduction studies in addition to conduction slowing. Myasthenic syndrome is a neuromuscular junction disorder and amyloidosis is associated with a form of axonal peripheral neuropathy; neither of these conditions would be likely to cause conduction block.

Which of the following may result from pistoning in a transtibial socket? A.Circumduction B.Dipping of the ipsilateral hip during swing phase C.Pain in the postero-distal position of the residual limb D.Hiking of the contralateral hip during swing phase

A. Circumduction 31.31% Pistoning of the residual limb within a prosthetic socket refers to the sliding of the limb into the socket during stance phase, and out of the socket during swing phase (specifically, terminal stance). This occurs due to a lack of suspension, such as wearing too few donning socks (insufficient ply) or having a hole in the suspension sleeve, thus breaking suction. As a result of the limb sliding into the socket, there is commonly pain at the inferior patella and antero-distal end of the limb from repetitively striking of the patellar tendon bar and distal end of the socket, respectively. When the amputee's weight is placed onto the prosthesis during early stance phase, the hip on that side will lower (e.g., dip) as they sink into the socket. As weight is transferred off the prosthesis during terminal stance, the limb will partly slide out of the socket, resulting in a relatively longer limb. To clear this limb during swing phase, the amputee with hike their hip on the prosthetic side and will often circumduct the prosthesis to avoid tripping over the foot.

A 5 year-old boy with spina bifida has full hip flexion against gravity and his knee extension strength is at least 4/5. The child has ankle dorsiflexion but not plantarflexion. His feet are in neutral position. Hip examination is symmetric. What will most likely be this child's primary means of mobility as he grows older? A.Community ambulation without assistive device B.Household ambulation with a cane C.Use of bilateral crutches for all mobility D.Independent manual wheelchair mobility

A. Community ambulation without assistive device 43.45% This child has a strong quadriceps muscle and no deformities noted at 6 months of age. He is reported to be healthy. Sitting balance and neurologic level were good predictors of ambulation potential. The best early predictor of ambulation in children with spina bifida is a strong quadriceps muscle. Negative predictors are spine and lower extremity deformities and obesity. Children do not typically learn to use crutches until 3 to 5 years of age or older.

A patient was involved in a high-speed motor vehicle collision with loss of consciousness and prolonged extrication from the vehicle. Head computed tomography (CT) scan was notable for a small subarachnoid hemorrhage. There have been several episodes of hypotension and hypoxemia since admission. What information in this clinical case makes diffuse axonal injury highly likely? A.High-speed motor vehicle collision B.Subarachnoid hemorrhage on head CT scan C.Episodes of hypoxia and hypotension D.Prolonged extrication from vehicle

A. High-speed motor vehicle collision 75.17% Diffuse axonal injury is most commonly seen after high-speed motor vehicle collisions and is the result of acceleration-deceleration and rotational forces. These traumatic forces result in widespread stretching of axons and axoplasmic dysfunction. DAI is the mechanism responsible for the loss of consciousness. Subarachnoid hemorrhage seen on CT scan is not necessarily due to a traumatic injury. Hypoxia and hypotension may occur in the setting of acute trauma and can cause anoxic injury to the brain. DAI is not worsened by prolonged extrication from the vehicle.

During Pre-swing in a normal individual's gait cycle, which muscle is most active? A.Ipsilateral iliopsoas muscle B.Ipsilateral tibialis anterior C.Contralateral gluteus maximus D.Contralateral gluteus medius

A. Ipsilateral iliopsoas muscle 34.65% The purpose of Pre-swing during the gait cycle is to prepare the limb to propel forward during swing phase. During this phase, the iliopsoas muscle of the ipsilateral limb is contracting concentrically to flex the hip for momentum. The contralateral gluteus medius muscle doesn't start to contract until initial swing, once the foot has left the ground, to stabilize the pelvis (prevent it from dropping). Neither the gluteus maximus or pretibial muscles are active during preswing; the ipsilateral gluteus maximus is eccentrically active only during loading response to prevent the trunk from rotating forward (from momentum), and the pretibial muscles do not start to contract until the foot leaves the ground during initial swing, to prevent foot drop.

Which factor is not correlated with life satisfaction following spinal cord injury? A.Level of injury B.Satisfaction with leisure activities C.Marriage D.Competitive employment

A. Level of injury 58.97% The highest levels of life satisfaction for individuals with spinal cord injuries correlated with employment, higher educational level, marriage, time since injury, satisfaction with leisure activities, social support, and adequate finances. Little correlation was noted between ratings of life satisfaction and level of injury, with respondents categorized as having paraplegia versus tetraplegia.

Which of the following is a treatment strategy for spatial neglect? A.Limb activated therapy B.Limit visual scanning C.Stimulating from the unaffected side D.Constraint induced movement therapy

A. Limb activated therapy 25.24% Limb activated therapy augmented with electrical stimulation via TENS unit is a treatment strategy for spatial neglect. Constraint induced movement therapy is a treatment for paresis of the paretic upper extremity in patients with motor impairment. Answers b and c are inappropriate treatments for spatial neglect since the goal would be to initiate awareness of the affect side.

A mother presents to your clinic with her 7-year-old boy. She states that her son is active in sports, and one of his coaches noted to her that he is "quite flat-footed" and should be evaluated. He does not complain of any pain or functional limitations. On exam you note that he has flexible bilateral pes planus feet, no pain on palpation, with a normal gait. You tell the mother: A.Most cases of "flat feet" are painless and do not cause problems. B.An MRI should be obtained to evaluate for tarsal coalition. C.Her son should use supportive footwear. D.Refer her son to a podiatrist for custom corrective orthotics.

A. Most cases of "flat feet" are painless and do not cause problems. 65.96% The majority of children with pes planus will develop an arch as they mature. Some children fail to do so but most do not have pain or limitations. At this time, there is no indication for intervention, alterations in footwear, or orthotics. Tarsal coalition may be a cause of pes planus in children, but this condition results in an inflexible foot; furthermore, the child is not experiencing pain. Some advocate walking/playing without footwear and engaging in intrinsic foot muscle strengthening exercises in order to help develop the arch, but there is little evidence to support this.

What test is used to assess for iliotibial band syndrome? A.Ober test B.McMurray's test C.Thomas test D.Anterior drawer test

A. Ober test 97.57% The Ober test is used to test for contraction of the iliotibial band. If a contracture of the tensor fascial lata or iliotibial band exists, then the leg remains abducted during the test. The leg may also remain abducted in polio or meningomyelocele. The Thomas test is used to detect a hip flexion contracture. McMurray's test is used to assess for a meniscal tear. Anterior drawer test is used to evaluate disruption of the anterior cruciate ligament.

A 70-year-old presents with hip pain 3 months after a total hip joint arthroplasty. Laboratory values reveal mildly elevated inflammatory markers (ie, ESR and CRP) without a leukocytosis. X-rays reveal a hybrid total hip prosthesis with a partially cemented femoral component with lucent zones at the proximal portion of the prosthesis-to-bone interface. The most likely etiology to account for the patient's pain and radiographic findings is: A.Polyethylene osteolysis B.Infection C.Trauma D.Hip girdle muscle weakness

A. Polyethylene osteolysis 78.97% Osteolysis in total joint replacements is usually caused by foreign body granulomatous reactions. Mechanical friction on polyethylene components abrade microscopic plastic particles into joint tissues. Ultimately, gross loosening of the prosthesis may occur. Infection is less likely in this case given the lack of a leukocytosis. There is no history of post-operative trauma. Hip girdle muscle weakness can contribute to the development of post-operative bursitis and tendinitis, but would not explain the patient's x-ray findings.

A 75-year-old female presents to clinic with a large, cool synovial effusion of her right shoulder. Physical examination is significant for pain-limited range of motion (ROM) and profound rotator cuff muscle weakness. X-rays of the patient's shoulder reveal superior subluxation of the humeral head, extensive bony destruction of the glenohumeral joint, soft-tissue effusion, and calcific deposits. The most likely diagnosis is: A.Pseudogout B.Lyme disease C.Osteonecrosis D.Charcot arthropathy

A. Pseudogout 37.08% The patient presents with a classic "Milwaukee shoulder" - MSS is a destructive, calcium phosphate crystalline arthropathy; it encompasses an effusion that is noninflammatory with numerous aggregates of calcium hydroxyapatite crystals in the synovial fluid, associated with rotator cuff defects . Calcium hydroxyapatite crystal disease is characterized by recurrent painful periarticular calcific deposits in tendons, soft tissues, or intra-articular surfaces. The pathophysiology is believed to be the intra-articular calcium hydroxyapatite deposition. MSS occurs in elderly patients typically aged 60-90 years. There is a female preponderance in the ratio of 4 : 1. Radiographic changes on plain X-ray show joint space narrowing, subchondral sclerosis with cyst formation, destruction of subchondral bone, soft-tissue swelling, capsular calcifications, and intra-articular loose bodies.Chronic musculoskeletal manifestations of Lyme disease may include bursitis and tendinitis, but not a destructive arthritis as depicted in the vignette. There was no history given alluding to a diagnosis of osteonecrosis of the humeral head or a neuropathic arthropathy (ie, Charcot arthropathy).

Which of the following factors predicted the highest risk for disability at one year following low back injury? A.Radicular symptoms B.Severity of injury C.Sedentary job D.Initial chiropractic treatment

A. Radicular symptoms 24.62% Per Official Disability Guidelines, "workers with more severe back injuries were more likely to be on disability after one year. Employees with pain spreading down into the leg indicating radiculopathy were at particularly high risk. Another significant predictor was the specialty of the first doctor seen after the back injury, and workers who saw a chiropractor were less likely to be disabled at one year. Certain job characteristics also affected disability risk, and risk was lower when employers offered modified duty. All of the significant factors were combined into a statistical model which was 88 percent accurate in identifying workers who would and would not be disabled after one year. In this study most of the psychological factors thought to contribute to chronic pain were not significant risk factors for disability. (Turner, 2008)"

A 63-year-old patient with amyotrophic lateral sclerosis presents to your clinic with insidious onset of dyspnea over the last several months. What is the most likely abnormality on pulmonary function testing? A.Reduced tidal volume B.Reduced lung tissue compliance C.Increased residual volume D.Increased forced expiratory volume

A. Reduced tidal volume 58.36% In restrictive lung disease, the primary abnormality is low tidal volume. In extrinsic restrictive lung disease (neuromuscular disease including amyotrophic lateral sclerosis, paralysis and kyphoscoliosis), this is due to weakness of the chest wall muscles. In intrinsic restrictive lung disease (pulmonary fibrosis, sarcoidosis), this is due to noncompliant lung tissue. In obstructive lung disease (asthma, COPD), abnormalities include increased residual volume and reduced forced expiratory volume due to incomplete exhalation or obstruction.

A patient complaining of weakness undergoes electrodiagnostic testing, which demonstrates normal nerve conduction studies. Needle electromyography is normal, except for variation in amplitude of individual motor units. What additional studies are indicated? A.Repetitive nerve stimulation studies B.Proximal conduction studies in upper extremities C.Somatosensory evoked potentials D.No further testing is indicated

A. Repetitive nerve stimulation studies 84.83% In a person with complaints of generalized weakness, differential diagnosis should include peripheral neuropathy, myopathy, NMJ disorders and motor neuron disorders. Normal nerve conduction studies exclude peripheral neuropathy. Otherwise normal EMG excludes myopathy and motor neuron disease. With variation in motor unit potentials neuromuscular junction disorders e.g. myasthenia gravis and Lambert Eaton syndrome and Botulism should be considered. Repetitive nerve simulation should be performed on distal and proximal muscles, a cold distal limb may produce false negative results. More than 10 % decrement between the first and the second response, followed by return to normal after a ten second maximum isometric contraction is in favor of MG whereas a 20-200% increment favors LEMS. Repetitive nerve stimulation produces mild detrimental response with reduced amplitude of the CMAP. Following exercise or high-frequency stimulation facilitation may be seen but varies from limb to limb.

A 23-year-old with C5 AIS-A tetraplegia admitted to inpatient rehabilitation was noted to have significant weight loss and vomiting after meals. X-rays with barium contrast show abrupt termination of the barium in the area of the duodenum. This clinical scenario is most likely related to A.Superior mesenteric artery syndrome B.Cholecystitis C.Chronic pancreatitis D.Neurogenic bowel

A. Superior mesenteric artery syndrome 78.62% Superior mesenteric artery (SMA) syndrome is seen mostly in patients with tetraplegia who present with abdominal distention, discomfort, and recurrent emesis following eating. It is caused by obstruction in the distal part of the duodenum as it passes behind the SMA and in front of the spine and aorta. It often occurs in patients who are immobilized and have lost a significant amount of weight and retroperitoneal fat. It is worse in the supine position and in patients who are in a body jacket. Upper GI series reveal an abrupt cessation of barium in the third part of the duodenum. Treatment includes sitting the patient in upright or positioning them in left side-lying after meals, nourishment to restore weight, and applying a lumbosacral corset to push the abdominal contents upward. Surgery is rarely indicated.

Which of the following finger deformities common in patients with rheumatoid arthritis is caused by synovitis of the metacarpophalangeal (MCP) or distal interphalangeal (DIP) joints creating a flexion contracture of the MCP joint, hyperextension of the PIP joint, and flexion of the DIP joint? A.Swan neck deformity B.Flexor tenosynovitis C.Boutonnière deformity D.Ulnar deviation

A. Swan neck deformity 89.97% A Boutonnière deformity is characterized by rupture of the terminal portion of the extensor hood and subluxation of the lateral bands causing the proximal interphalangeal (PIP) joint to protrude through the split tendon as if it were a button hole. In contrast to a Boutonnière deformity, a Swan neck deformity is caused by synovitis of the metacarpophalangeal (MCP) or distal interphalangeal (DIP) joints creating a flexion contracture of the MCP joint, hyperextension of the PIP joint, and flexion of the DIP joint. Flexor tenosynovitis commonly affects the extensor pollicis brevis (EPB) and abductor pollicis longus (APL) tendons (ie, DeQuervain's tenosynovitis) causing radial-sided wrist pain. Ulnar deviation of the fingers in patients with rheumatoid arthritis is secondary to unrestrained pull of the ulnar finger flexors.

With which of the following is polymyalgia rheumatica most commonly associated? A.Temporal arteritis B.Rheumatoid arthritis C.Statin-induced myositis D.Spondyloarthritides

A. Temporal arteritis 92.56% No pathogenic associations with polymyalgia rheumatica (PMR) have been confirmed. 5-15% of patients with PMR will have temporal arteritis.

The Galveston Orientation and Amnesia Test (GOAT) and Orientation Log (O-Log) are standard measures for assessing post-traumatic amnesia (PTA). A patient is no longer in PTA when: A.The O-Log score is 25 or higher for 2 consecutive days B.The O-Log score is 75 or higher for any 2 days in the same week C.The GOAT score is 25 or higher for 2 consecutive days D.The GOAT score is 75 or higher for any 2 days in the same week

A. The O-Log score is 25 or higher for 2 consecutive days 61.17% The GOAT score can range from 0 to 100, with a score of 75 or better defined as normal. The end of PTA can be defined as the date when the patient scores 75 or higher in GOAT for two consecutive days. The O-Log score can range from 0 to 30, with a score of 25 or better on two consecutive days defined as the end of PTA.

Juvenile idiopathic arthritis (JIA) can be differentiated radiographically from ankylosing spondylitis (AS) by the: A.Constellation of cervical facet joint ankylosis and vertebral body hypoplasia in patients with JIA B.Decreased vertebral body disc height in the cervical spine of patients with AS C.Prevalence of radiographic findings in the lumbar spine of patients with JIA D.Presence of enthesopathy and tarsal disease in the first year of presentation in patients with AS

A. Constellation of cervical facet joint ankylosis and vertebral body hypoplasia in patients with JIA 43.77% By definition, JIA affects the immature, growing skeleton. Diffuse ankylosis of the cervical facet joints in the setting of vertebral body and disc hypoplasia is virtually diagnostic. In contrast, AS affects the mature skeleton after the intervertebral discs have formed. The cervical spine is more commonly involved in patients with JIA than other spinal levels with AA instability being the most common radiographic finding. Enthesopathy and tarsal disease in the first year of presentation is diagnostic for JIA even in the absence of axial disease. Klippel-Feil syndrome may be difficult to distinguish radiographically from JIA unless posterior element fusion is a dominant feature in the cervical spine.

A 36-year-old is referred for electrodiagnostic testing. The following studies were obtained: (Editorial: Unable to upload pics, but NCS: increased latency L median motor and sensory, No Response right median motor and sensory; ulnar studies normal; EMG: reduced recruitment, PSW, fibs in bilateral APB) -TG The most likely diagnosis is: A.Bilateral median mononeuropathy at the wrist B.Distal peripheral polyneuropathy C.Lower cervical radiculopathy D.Bilateral ulnar mononeuropathy at the wrist

A.Bilateral median mononeuropathy at the wrist 88.75% The latency, abnormalities are confined to both median nerves. Both sensory and motor fascicles are affected. Since ulnar nerve conduction studies are normal, peripheral neuropathy is excluded. As normal paraspinals and other C8 innervated muscles are normal, radiculopathy is unlikely. Other proximal median muscles are normal indicating pathology localized to wrist level. On the right side, no volitional motor units were visualized, indicating poor prognosis.

Which of the following is a provocative test for sacroiliac joint? A.Gaenslen's test B.Ortolani test C.Log Roll test D.Straight leg test

A.Gaenslen's test Fortin Finger, Patrick's, Ischial Compression, Gaenslen, and pubic compression tests are all known maneuvers for provocation of sacroiliac joint pain. Three positive maneuvers have a specificity of 78% for SIJ related pain

A 25-year-old is brought to the emergency department following a motor vehicle collision. On examination, she can open her eyes when spoken to and is able to follow commands. Although disoriented, she is able to converse with you. What is her Glasgow Coma Score (GCS)? A.12 B.13 C.14 D.15

B. 13 54.71% The GCS is scored between 3 and 15, 3 being the worst, and 15 the best. It is composed of 3 parameters: Best Eye Response, Best Verbal Response, Best Motor Response, as given below. In this case, the GCS score is 3+4+6=13 (PIcture of GCS scale not included, look it up ya dingus)

An 18-year-old female presents with a sudden onset of posterior thigh pain that occurred while water skiing over a week ago. She reports a "pop" may have been perceived at the onset of the pain. Ecchymosis is noted in the posterior thigh. Pain is noted with straight leg raise. X-ray of the thigh is negative for fracture. What is the most likely diagnosis? A.An avulsion fracture B.Acute hamstring strain C.ACL tear D.Hip dislocation

B. Acute hamstring strain 83.89% The patient has an acute hamstring strain. Unless an avulsion fracture with bony fragment or apophyseal fracture is suspected, plain x-rays are of little use in the work up of an acute hamstring injury. Ultrasonography (US) and MR imaging technologies are utilized instead. US is extremely accurate in the acute phase. However, MRI is most commonly used.

What medical condition would demonstrate the feature shown in this motor nerve conduction study? Figure demonstrates temporal dispersion of bilateral tibial motor studies A.Acute motor axonal neuropathy (AMAN) B.Acute inflammatory demyelinating polyradiculoneuropathy (AIDP) C.Diabetic polyneuropathy D.Mononeuropathy Multiplex (MNM)

B. Acute inflammatory demyelinating polyradiculoneuropathy (AIDP) 46.28% The tracings above demonstrate temporal dispersion, which is pathognomonic for an acquired demyelinating disease, and is among the diagnostic criteria for acute inflammatory demyelinating polyradiculoneuropathy (AIDP), chronic inflammatory demyelinating polyradiculoneuropathy (CIPD), multifocal motor neuropathy (MMN), multifocal acquired demyelinating sensory and motor (MADSAM) neuropathy or Lewis-Sumner syndrome, and distal acquired demyelinating symmetric (DADS). Demyelination is not a feature of acute motor axonal neuropathy (AMAN), therefore temporal dispersion is not pathognomonic. Diabetic polyneuropathy and mononeuropathy multiplex are axon loss neuropathies.

Cardiopulmonary exercise testing is used to evaluate: A.Cardiac ischemia B.Aerobic capacity C.Stroke volume D.Muscle strength

B. Aerobic capacity 85.71% While estimates can be made of cardiac stroke volume, muscle strength and sprint velocity, aerobic capacity is what is measured by cardiopulmonary exercise testing. Cardiac ischemia may be detected by the 12-lead monitoring used on cardiopulmonary exercise testing (CPET), but CPET is a functional test, not a diagnostic test like cardiac exercise testing or the variants such as thallium, persantine, or other tests.

A 12-year-old ballerina presents to your clinic with right ankle pain. She was dancing pain-free until this year when she began to dance on pointe. On examination, you note tenderness to palpation at the posterior talocrural joint, pain with end passive ankle plantarflexion range of motion, and no pain with resisted active ankle dorsiflexion or plantarflexion. Which of the following would you anticipate with imaging? A.Tendinosis of the Achilles tendon on ultrasound B.An os trigonum on ankle radiographs C.A Haglund's deformity on ankle radiographs D.A split peroneus longus tendon on MRI

B. An os trigonum on ankle radiographs 65.35% Dancers who go on pointe are at risk for posterior ankle impingement syndrome, which results from impingement of the posterior aspect of the talus between the tibia and calcaneus with repetitive ankle plantarflexion. One predisposing factor for this condition is the presence of an os trigonum, which results from an accessory ossification center of the posterior talus. While Achilles tendinopathy and peroneal tendon injuries are not uncommon in ballet dancers, the patient's location of pain and physical examination are not indicative of either of these as the diagnosis. Similarly, her exam is not suggestive of Haglund's deformity; furthermore, this patient would be relatively young for this condition.

A 67-year-old is walking his dog and falls on the sidewalk. He is found to have a cervical spinal cord injury with greater weakness in his upper limbs than his lower limbs. Sensation to pinprick and light touch is impaired below the level of injury. This spinal cord syndrome is most likely: A.Posterior cord syndrome (Tabes Dorsalis) B.Central cord syndrome C.Brown-Sequard syndrome D.Anterior cord syndrome

B. Central cord syndrome 92.23% Central cord syndrome was first described by Schneider in 1954 and is characterized by "disproportionally more motor impairment of the upper than lower extremity, bladder dysfunction, urinary retention, and varying degrees of sensory loss below the level of the lesion." This spinal cord syndrome is the most common of the spinal cord syndromes making up approximately 9% of all traumatic spinal cord injuries (SCI). This syndrome is classically associated with geriatric patients with cervical spondylosis who sustain a hyperextension injury. It is generally thought to have a favorable prognosis as compared to other SCI syndromes particularly when this syndrome affects younger patients.Brown-Sequard syndrome is a lesion that causes ipsilateral motor and proprioceptive loss and contralateral loss to pain and temperature below the level of the lesion. Anterior cord syndrome is a lesion that affects the anterior 2/3 of the spinal cord while preserving the posterior columns. It is characterized by motor paralysis with hyperesthesia, hypoalgesia below the level of the lesion with preservation of touch, proprioception and vibratory sense. It is associated with flexion injuries and vascular insufficiency produced by occlusion of anterior spinal artery.

Which of the following is true regarding running and the gait cycle? A.Stance and swing phases each occur for 50% of the cycle B.Double stance does not occur C.Initial contact occurs exclusively with the heel D.Center of gravity is at its highest point during midstance

B. Double stance does not occur 78.62% Running is the bipedal locomotion at a cadence sufficient such that a period of double stance does not occur. Double stance is replaced by a period of double limb float, when both limbs are in swing phase. As opposed to normal (walking) gait, stance phase occurs for 40% of the gait cycle, while swing phase occurs for 60%. Additionally, the center of gravity reaches its lowest point in the cycle during midstance, whereas this is its highest point during walking. Initial contact can occur with either the heel or forefoot, as is common in sprinters, or those running uphill.

Which is the most common neurological dysfunction after a liver transplant? A.Seizures B.Encephalopathy C.Stroke D.Depression

B. Encephalopathy 86.08% In a study by Ghaus et al, 62% of liver transplant patients developed encephalopathy. Seizures occurred in 11% and stroke in 9%. In another study by Rothenhausler, 3% of transplant patients had depression.

What is the most common cause of emergency room visits for traumatic brain head injuries? A.Motor Vehicle Collisions B.Falls C.Gun shot wounds D.Struck by an object

B. Falls 33.13 In 2014, falls became the leading cause for TBI. Falls account for 48% of all TBI related emergency department room visits. Falls disproportionately affect children and older adults. Being struck by or against an object is the second leading cause of TBI-related emergency department visits. Falls and motor vehicle crashes were the leading causes of all TBI-related hospitalizations.

You are a physician volunteering at a marathon medical tent and a 30-year-old female presents at the finish line with confusion and dizziness wearing a shirt that feels very damp. She says that he has been drinking water at regular intervals and has been urinating frequently. Which of the following is the most likely cause of her symptoms? A.Postural Hypotension B.Hyponatremia C.Frostbite D.Dehydration

B. Hyponatremia 76.38% Exercise associated hyponatremia is defined by a serum or plasma sodium concentration less than 135 mmol/L but one usually will get symptoms when sodium levels drop below 130. Early signs and symptoms include nausea, vomiting and headaches. As the severity of exercise associated hyponatremia develops, a patient may experience symptoms of altered mental status, seizures, respiratory distress, coma and death as a development of worsening cerebral edema. Risk factors for exercise associated hyponatremia are low body weight, female sex, four hours exercise duration, slow running or performance pace, race inexperience, excessive drinking behavior, high availability of drinking fluids, and extreme hot or cold environmental conditions. In this case the patient is most likely suffering from hyponatremia due to excessive drinking.

A 26-year-old is admitted to the inpatient rehabilitation unit following a traumatic brain injury. They are confused, inappropriate, agitated, and requires a padded floor bed. What is their Rancho Los Amigos level? A.III B.IV C.V D.VI

B. IV 81.23% The Rancho Los Amigos level IV applies to persons who are confused and agitated. The need of a padded floor bed indicates that the patient is restless and agitated. This scale is a useful descriptive tool for clinicians and family members to follow a patient's cognitive and behavioral functioning on an eight-point scale.

Clinical symptoms and signs of tethered cord syndrome include: A.Weakness in upper extremities B.Increase in scoliosis C.Loss of reflexes in lower extremities D.More severe headaches

B. Increase in scoliosis 23.45% The most common clinical signs or symptoms of a tethered cord include spasticity in the lower extremities, decline in lower extremity strength, and worsening scoliosis. Other signs and symptoms that strongly suggest tethering of the spinal cord include back pain, changes in urologic function, changes in gait, changes in sensation, and development of lower extremity contractures.

A patient presents with right posterolateral hip pain radiating into the lateral leg for 2 months, and has focal weakness of right great toe extension. Electrodiagnostic studies show the following. (Editorial: NCS: Slight side-to-side R<L amplitude difference with tibial and peroneal, normal latencies; nl sural and sup peroneal sens bl. EMG: Red Recr, Fibs/PSWs R SH Biceps, Tib ant, peroneus longus, glut med, Lumb paraspinal; normal vastus, semimembranosus, medial gastroc) -TG These findings are most consistent with right: A.Lumbosacral plexopathy B.L5 radiculopathy C.Sciatic neuropathy D.Common peroneal neuropathy

B. L5 radiculopathy 89.67% L5 is the most common unilateral/unilevel radiculopathy observed clinically. The clinical exam suggests weakness of the Extensor hallucis longus (EHL) that is supplied by the deep peroneal nerve mainly through the L5 root.The normal sensory studies make it less likely that this is a plexopathy or peripheral nerve injury. The tibial and peroneal motor responses are in the normal range but the right tibial CMAP is found in the common peroneal and superior gluteal nerve distribution. Finally, the involvement of the lower lumbar paraspinal muscles is consistent with a lumbar radiculopathy.

Which clinical test is most sensitive and specific for an anterior cruciate ligament tear? A.Pivot shift B.Lachman C.Anterior drawer D.Posterior drawer

B. Lachman 68.09% The pivot shift test, Lachman test, and anterior drawer test are all clinical tests for anterior cruciate ligament (ACL) tears. The posterior drawer test is a clinical test for posterior cruciate ligament (PCL) tears. The Lachman test is the most sensitive and specific test for the diagnosis of an ACL tear, particularly with acute injuries. The Lachman test is performed with the knee positioned at approximately 15o of flexion. The femur is stabilized with one hand while the other hand applies pressure to the posterior aspect of the proximal tibia and the examiner attempts to translate the tibia anteriorly

Abnormalities on needle EMG in which of the following muscle pairs can confirm the presence of a posterior cord brachial plexopathy and exclude a radial neuropathy? A.Triceps/brachioradialis B.Latissimus dorsi/Deltoid C.Serratus anterior/Rhomboid D.Biceps/serratus anterior

B. Latissimus dorsi/Deltoid 76.6% In a posterior cord brachial plexopathy the proximal and distal radial innervated muscles (EIP, ECRL, Brachioradialis and Triceps) plus the deltoid and latissimus dorsi will be affected. Biceps is innervated by the musculoctaneous nerve and lateral cord of the brachial plexus. Serratus anterior is innervated by the long thoracic nerve arising from the C5, C6 and C7 roots. Rhomboid is innervated by dorsal scapular nerve, C5 root.

A 29-year-old patient suffers from a longstanding complete paraplegic spinal cord injury. She presents to the outpatient office complaining of ascending numbness and paresthesias. She reports that this has been insidious and slowly worsening over the course of months. Physical exam yields new sensory deficits above the previous sensory level. The most appropriate diagnostic test in this setting would be: A.EMG B.MRI with and without contrast C.CT scan with and without contrast D.Cervical myelogram

B. MRI with and without contrast 89.66% The patient is presenting with history and physical findings consistent with syrinx. Syrinx is an abnormal cystic collection of fluid within the spinal cord. Though it may be idiopathic it is often associated with previous trauma and spinal cord injury. MRI is the modality of choice in diagnosing syrinx. CT myelography may be useful when MRI is contraindicated or unavailable.

Which of the following aids in prognosticating outcomes of TBI at 6 months post injury? A.Transcranial magnetic stimulation B.Magnetic resonance spectroscopy C.Apolipoprotein E D.Positron emission tomography

B. Magnetic resonance spectroscopy 58.05% Magnetic resonance spectroscopy (MRS) provides information about the neurochemical status of the brain and may indicate the extent of brain damage; therefore, it provides important data for evaluating expected outcome. Studies found that MRS studies correlated with outcomes 6 months later. Furthermore, several studies found threshold levels for the metabolites that also correlate with prognosis. Positron emission tomography (PET) uses radioactive tracers that localize to metabolically active regions. PET scanning is more sensitive in detecting abnormalities after traumatic brain injury than is standard neuroimaging, although PET has poor specificity and has not been studied in regard to prognostic value. Earlier studies suggest that the presence of apolipoprotein-E4 allele (APOE-e4) would indicate a worse functional outcome, but that finding has not been replicated. There are conflicting reports on APOE-e4. It is unlikely that a patient's APOE-e4 status will play a role in prognostication at the present time. Transcranial magnetic stimulation is useful in examining cortical motor representation after cerebral injury and may be useful in treatment of TBI but has not been studied in regard to prognostic value.

Which electrodiagnostic test is most useful in evaluating a patient suspected of having a myopathy? A.Late motor responses B.Needle electromyography C.Motor nerve conduction studies D.Repetitive stimulation

B. Needle electromyography 94.83% Needle electromyography is the most useful electrodiagnostic test in the assessment of patients with possible myopathy. Most nerve conduction studies are normal in patients with myopathy. The needle examination can show characteristic changes in the motor unit action potential duration, morphology, amplitude and recruitment. Furthermore, it can be very helpful in determining the distribution of the muscles involved.

Which electrodiagnostic finding is most compatible with early stage anterior horn cell disease? A.Small or absent sensory responses B.Normal motor conduction studies C.Small amplitude motor unit potentials D.Prolonged somatosensory evoked potentials

B. Normal motor conduction studies 57.45%In anterior horn cell diseases such as amyotrophic lateral sclerosis (ALS), the sensory nerves are not usually affected. In the early stages of the condition motor nerve conductions can be normal. Because ALS is a neuropathic condition with ongoing denervation and reinnervation, one would not expect to find small amplitude motor units with early or increased recruitment; that pattern represents a myopathic process.

Which measure evaluates patients' self-perception of disability specifically for low back pain? A.Fear-avoidance beliefs questionnaire (FABQ) B.Oswestry Disability Index (ODI) questionnaire C.Minnesota multiphasic personality inventory (MMPI) D.Short Form 36 (SF-36)

B. Oswestry Disability Index (ODI) questionnaire 77.59% ODI is used to track patient perception of disability over the course of treatment, specifically for low back pain. In addition to being used as an outcome measure in research, physician offices are also utilizing this tool for outcome tracking.

Which clinical factor correlates most powerfully with outcome after a traumatic brain injury? A.Age at injury B.Posttraumatic amnesia C.Length of Coma D.Imaging findings at Injury

B. Posttraumatic amnesia 51.37% Age, length of coma, and duration of posttraumatic amnesia (PTA) all provide valuable information that the clinician can use to mark milestones, but the most powerful of these is the duration of PTA. The longer the duration of the PTA, the worse the outcome. It is unlikely for a person with PTA lasting less than 2 months to have a serious disability; however, the likelihood of a good recovery is poor if the duration of PTA extends beyond 3 months. The use of neuroimaging to date has not been clinically helpful. MRI identifies lesions more frequently than computed tomography. Conventional MRI findings that strongly predict outcome include diffuse axonal injury (DAI), total lesion burden and DAI in the brainstem. However, the absence of lesions on MRI does not rule out TBI, the cost and difficulty of the exam in agitated or medically sick patients, and the lack of good research data preclude the routine clinical use of MRI for prognostic purposes.

Which of the following factors predict a poor prognosis for return to work following low back injury? A.Higher educational level B.Pre-injury heavy manual job C.Age less than 45 years old D.High life and job satisfaction

B. Pre-injury heavy manual job 96.96% Per Official Disability Guidelines, "Non-medical factors identified as being potentially associated with poorer return to work outcomes include: low worker educational levels, a pre-injury heavy manual job, older than 45 years, smoking, positive Waddell's signs, less life satisfaction, receiving compensation, workplace climate (including bullying), prior absence history, abuse of drugs and alcohol, long commuting distance to/from work, attorney involvement, long hours worked, work overload and pressure, high unpaid workload (particularly with women), obesity, and blaming others for the injury. (Mills, 2008)

You are asked to perform electrophysiologic studies on a 21-year-old man recently treated for testicular cancer with cisplatin who developed bilateral upper and lower extremity pain and paresthesias. Nerve conduction studies demonstrate low sensory nerve action potential (SNAP) amplitudes in the median, ulnar, and radial nerves bilaterally but normal SNAP amplitudes in the lower extremities. Compound muscle action potential (CMAP) amplitudes are normal in both the upper and lower extremities. Needle electromyography (EMG) is normal. What is the most likely cause of the patient's symptoms? A.Brachial plexopathy B.Sensory ganglionopathy C.Polyradiculopathy D.Acquired idiopathic demyelinating polyradiculoneuropathy (AIDP)

B. Sensory ganglionopathy 74.47% This patient most likely has a sensory ganglionopathy from exposure to platinum-based chemotherapy (cisplatin) used to treat his testicular cancer. As opposed to neurotoxic chemotherapeutics such as the vinca alkaloids and taxanes which cause a length-dependent axonopathy, platinum analogues exert their putative neurotoxic effect by intercalating in the DNA of the dorsal root ganglion thereby killing or disrupting function of affected sensory nerves. Sensory neuropathy caused by platinum analogues is not length dependent, so it is not unusual to see the sensory amplitudes in the upper extremities more affected than those in the lower extremities. Because platinum analogues do not cross the blood brain barrier to affect the anterior horn cells at contemporary doses, the CMAP amplitudes and needle EMG should be normal.

What is the most common etiology of low back pain in a 13-year-old female athlete? A.Disc abnormalities B.Spondylolysis C.Muscle-tendon strain D.Osteoarthrtitis

B. Spondylolysis 65.65% n a study of 100 adolescents with low back pain, spondylolysis of the pars interarticularis were present in 47% of adolescents, 11% of the adolescent group had back pain attributable to disc abnormalities, while only 6% of adolescents were diagnosed as having muscle-tendon strain. Spinal stenosis and osteoarthritis were not encountered in the children.

A patient presents to your clinic complaining of unilateral left head and neck pain that he describes as stabbing and radiating from the left side of the neck to the left frontal region. It worsens with turning his head to the left. You suspect cervicogenic headaches. Which of the following do you expect to find on physical examination? A.Positive Spurling sign B.Tenderness of the upper cervical facet joints C.Diminished reflex of the ipsilateral biceps brachii D.Shock-like pain radiating down the axial spine with cervical flexion

B. Tenderness of the upper cervical facet joints 82.07% Cervicogenic headaches are typically associated with pain and tenderness emanating from the upper cervical facets. A positive Spurling sign is radiating, lancinating pain in the upper extremity with ipsilateral extension, rotation, and side bending of the head. This test is associated with cervical nerve root compression. A diminished biceps brachii reflex can occur in a C5-6 radiculopathy. Shock-like pain radiating down the axial spine with cervical flexion is known as "Lhermitte's sign." It suggests a lesion of the dorsal columns of the spinal cord, and is classically associated with multiple sclerosis, though it can occur with many upper motor neuron disorders.

In a 12-year-old with severe traumatic brain injury, which of the following best predicts outcomes 1 year after injury? A.Initial Glasgow Coma Score B.Time to follow commands C.Duration of post-traumatic amnesia D.Focal magnetic resonance imaging findings

B. Time to follow commands 4.85% Time to follow commands significantly contributes to global functional outcomes at 1- to 2-year follow-up. There is a trend toward additional predictive power with post-traumatic amnesia. Motor GCS at 72 hours is more predictive of outcomes than initial GCS. Larger total volume of MRI lesions and greater depth are worse predictors of outcomes.

An 18-year-old baseball pitcher presents with posterior elbow pain that is worse during throwing. A differential diagnosis for the patient's elbow pain includes: A.Flexor-pronator muscle tendinopathy B.Trochlear chondromalacia C.Ulnar collateral ligament insufficiency D.Distal biceps tendonitis

B. Trochlear chondromalacia 38.3% The differential diagnosis for posterior elbow pain in a thrower includes: trochlear chondromalacia, olecranon osteophytes, olecranon stress fracture, and triceps tendinitis. Flexor-pronator muscle tendinopathy and distal biceps tendinitis would cause anterior elbow or proximal forearm pain, and ulnar collateral ligament insufficiency would be associated with medial elbow pain.

A patient complains of right upper limb weakness of 6 months duration. Motor exam reveals marked atrophy of first dorsal interosseous muscle. Muscle tone, reflexes and sensation are normal. There is no history of any other comorbidity. (Editorial: NCS normal median, ulnar to D5 and DUC sensory; Median motor and ulnar motor to ADM normal; ulnar motor to FDI with NR on right. EMG: Right FDI with abn spont activity and no voluntary activation) -TG What is the most likely diagnosis? A.Ulnar neuropathy at the elbow B.Ulnar neuropathy at the wrist C.C8 radiculopathy D.Medial cord brachial plexopathy

B. Ulnar neuropathy at the wrist 82.67% Differential diagnosis of painless hand weakness with atrophy of FDI includes motor neuron disease, focal spinal cord lesion and a lesion of purely motor branch of the ulnar nerve. Normal median sensory conduction studies rule out median mononeuropathy. Normal ulnar sensory conduction studies rule out a lesion of ulnar nerve at the elbow. Intact dorsal ulnar cutaneous nerve study further localizes this problem to the wrist level as this nerve branches off prior to the wrist. Ulnar nerve CMAP from first dorsal interosseous muscles showed no response. This localizes the lesion to the deep ulnar branch. The ulnar nerve proceeds through the distal forearm and supplies the dorsal and palmar cutaneous branches, then enters the Guyon's canal. Within the canal it divides into superficial and deep branches. The superficial branch supplies sensation to the volar aspect of the fifth and medial fourth digits as well as the palmaris brevis muscle. The deep palmar branch supplies the remaining hypothenar muscles proximally (abductor digiti minimi, flexor digitiminimi and opponens digiti minimi) and remaining muscles distally (third and fourth lumbricals, the four dorsal and three palmar interossei, the adductor pollicis, and the deep headof the flexor pollicis brevis). This pattern of ulnar neuropathy at the wrist is the pure motor deficits caused by compromise of the deep branch distally.

A child with oligoarticular juvenile idiopathic arthritis should have regular evaluation of which system: A.Hearing B.Vision C.Skin D.Endocrine

B. Vision 82.52% Iridocyclitis and silent uveitis are common in children with oligoarthritis and regular evaluation by an ophthalmologist is recommended.

A patient who had a right MCA stroke presented one hour after symptom onset and received intravenous thrombolysis with t-PA, improving his NIHSS from a 12 to a 5. He is ambulating with physical therapy on the acute medical floor. Which factor has greatest evidence to predict his chances of going home? A.Premorbid functional status B.t-PA within the three hours of symptoms C.Early mobilization within two days D.Anatomic location of stroke.

B. t-PA within the three hours of symptoms 34.65% Patients who receive intravenous thrombolysis for acute ischemic stroke are more likely to be discharged directly home after hospitalization with outpatient neurorehabilitation therapies. There is no correlation with the exact time within the three-hour window. His premorbid functional status, location of stroke, and early mobilization are important components of his rehabilitation; however, they have no bearing on the efficacy of t-PA therapy.

Lumbar discography is primarily utilized for the assessment of: A.Intradiscal infection B.Concordant discogenic pain C.Schmorl's nodes D.Arterial supply

B.Concordant discogenic pain 78.28% Lumbar discography is a controversial diagnostic procedure due to concerns about its clinical utility and safety. The most common indication for discography is for patients with persistent back pain in whom a disc abnormality is suspected as the pain generator. The most critical aspect of discography is whether intradiscal pressurization reproduces the patient's concordant back pain. Since the accuracy of the test depends on the patient's subjective pain response, it is more useful when adjacent discs do not elicit a similar pain response because they serve as negative controls. Common criteria for diagnosis of a painful, internally disrupted disk include a pain level of 7 or higher, concordant pain reproduction at a pressure less than 50 mmHg above opening pressure, annular tear grade III or higher, injected contrast volume of 3.5 mL or less, and a pain-free control disk at an adjacent level.

Abnormalities in which of the following electrodiagnostic tests can be used to distinguish a medial cord brachial plexopathy from a proximal ulnar neuropathy? A.First dorsal interosseous needle EMG B.Medial antebrachial cutaneous sensory NCS C.Flexor digitorum profundus (to 5th digit) needle EMG6 D.Lateral antebrachial cutaneous sensory NCS

B.Medial antebrachial cutaneous sensory NCS78.42% The Medial antebrachial cutaneous nerve arises from the medial cord of the brachial plexus and the Medial antebrachial cutaneous sensory study assesses the integrity of the medial cord and the lower trunk. The Lateral antebrachial cutaneous nerve arises from the lateral cord of the brachial plexus and the Lateral antebrachial cutaneous sensory study assesses the integrity of the lateral cord and the upper trunk. The first dorsal interosseous and flexor carpi ulnaris are innervated by the ulnar nerve and will be affected in an ulnar neuropathy.

A 35-year-old Caucasian female with a history of lupus presents to clinic with bilateral shoulder girdle muscle aching pain and weakness and normal x-ray. She was treated with corticosteroids and an antimalarial drug six months ago. Laboratory values reveal a normal creatine phosphokinase (CPK) level, ESR, and CRP. Which of the following best explains the patient's clinical symptoms? A.Humeral head osteonecrosis B.Steroid-induced myopathy C.Polymyalgia rheumatica D.Rotator cuff tendon tear

B.Steroid-induced myopathy63.83% Patients with SLE can develop myopathy as a consequence of corticosteroid or antimalarial use. Overt myositis with elevations of CPK occurs in < 15% of patients. Electromyography and muscle biopsy findings may be normal or consistent with an inflammatory myopathy such as dermatomyositis or polymyositis. Although osteonecrosis in SLE is frequently bilateral, it is not necessarily simultaneous, and usually presents with persistent pain localized to a single joint. In contrast to SLE, patients with polymyalgia rheumatica are older, have elevated acute phase reactants, and respond remarkably well to low doses of corticosteroids. There is no history to suggest a rotator cuff tendon tear.

The modified Ashworth score corresponding to increased muscle tone with difficult passive movement is: A.1+ B.2 C.3 D.4

C. 3 83.17% The modified Ashworth Scale (MAS) is used to score passive individual muscle movement as the limb segment is moved through its entire arc of joint motion over a one-second time period. The amount of non-volitional resistance the examiner encounters is quantified to indicate the intensity of spasticity: 0 No increase in muscle tone 1 Slight increase in muscle tone (usually a catch and release feeling) 1+Slight increase in muscle tone with minimal resistance throughout remainder of motion 2Marked increase in muscle tone but easily moved 3 Increased muscle tone with difficult passive movement 4Rigid without movement.

In the severe TBI population, the possibility of good recovery is unlikely when post-traumatic amnesia (PTA) exceeds: A.1 month B.2 months C.3 months D.6 months

C. 3 months 51.78% Lower GCS scores are associated with worse outcome. Severe disability is unlikely when length of coma is < 2 weeks and good recovery is unlikely when coma > 4 weeks. Severe disability is unlikely when PTA is < 2 months and good recovery is unlikely when PTA > 3 months. Good recovery is unlikely when patient is >65 y/o. Good recovery is unlikely when bilateral brainstem lesions are present on early MRI.

A hinged/articulated AFO with plantar flexion stop and free dorsiflexion would best be prescribed in which of the following scenarios? A.A patient with flaccid foot drop B.To improve function of patient with a crouched gait pattern C.A patient with some dorsiflexion strength and mild plantarflexion spasticity D.To attempt to correct rigid ankle deformities

C. A patient with some dorsiflexion strength and mild plantarflexion spasticity 73.56% Hinged or articulated AFOs with plantar flexion stop is indicated when there is sufficient hindfoot dorsiflexion and strength that will allow movement of tibia over the foot in stance. They may be used in the presence of milder hypertonia or spasticity. It may promote crouched posture by allowing increased dorsiflexion and knee flexion, therefore option B is incorrect. Option D describes a solid ankle AFO and its indications.

Which factor is a criterion for hip osteoarthritis? A.Periarticular osteopenia B.Femoral head erosions with sclerosis C.Acetabular osteophytes D.Erythrocyte sedimentation rate above 20mm/hr

C. Acetabular osteophytes 70.34% The American College of Rheumatology states that the criteria for osteoarthritis of the hip are hip pain along with two of the three findings: erythrocyte sedimentation rate less than 20mm/hr, radiographic evidence of femoral/acetabular osteophytes, or radiographic evidence joint-space narrowing. Periarticular osteopenia and femoral head erosions are more likely to be seen in patients with rheumatoid arthritis.

How should the electrodiagnostic study shown below be BEST characterized? Figure 1 - (Ulnar motor nerve conduction studies) Very low amplitude CMAP with above elbow stimulation, normal appearing CMAPs with wrist and below elbow stimulation. Figure 2 - (Needle EMG, first dorsal interosseous) Appears to show some sharps and fibs A.Temporal dispersion B.Neurapraxic conduction block C.Axonal degeneration D.Demyelination

C. Axonal degeneration 28.8% Editorial: Motor NCV is consistent with conduction block however in a pure demyelinating condition with conduction block, reduced recruitment would be the only finding on needle EMG. -TG Conduction block is the failure of an action potential to propagate past a particular point in the nervous system whereas conduction is possible below the point of the block. Documented by demonstration of a reduction in the area of a compound muscle action potential greater than that normally seen with stimulation at two different points on a nerve trunk; anatomic variations of nerve pathways and technical factors related to nerve stimulation must be excluded as the cause of the reduction in area.Temporal dispersion is a relative desynchronization of components of a compound muscle action potential due to different rates of conduction of each synchronously evoked component from the stimulation point to the recording electrode. It may be due to normal variability in individual axon conduction velocities, especially when assessed over a long nerve segment, or to disorders that affect myelination of nerve fibers.Axonal degeneration is the degeneration of the segment of a nerve distal to the cell body with preferential distal pathology. It is impossible to distinguish axonal degeneration from conduction block using nerve conduction studies alone, particularly in the first 7-10 days after nerve injury. A neurapraxic lesion would be characterized by conduction block, whereas an axonotmetic or neurotmetic lesion would demonstrate axonal loss, both of which initially appear similar on nerve conduction studies. After 7-10 days, however, the process of Wallerian axonal degeneration would then manifest as findings on needle EMG.

Which of the following finger deformities common in patients with rheumatoid arthritis is characterized by rupture of the terminal portion of the extensor hood and subluxation of the lateral bands? A.Swan neck deformity B.Flexor tenosynovitis C.Boutonnière deformity D. Ulnar deviation

C. Boutonnière deformity 65.7% A Boutonnière deformity is characterized by rupture of the terminal portion of the extensor hood and subluxation of the lateral bands causing the proximal interphalangeal (PIP) joint to protrude through the split tendon as if it were a button hole. In contrast to a Boutonnière deformity, a Swan Neck deformity is caused by synovitis of the metacarpophalangeal (MCP) or distal interphalangeal (DIP) joints creating a flexion contracture of the MCP joint, hyperextension of the PIP joint, and flexion of the DIP joint. Flexor tenosynovitis commonly affects the extensor pollicis brevis (EPB) and abductor pollicis longus (APL) tendons (ie, DeQuervain's tenosynovitis) causing radial-sided wrist pain. Ulnar deviation of the fingers in patients with rheumatoid arthritis is secondary to unrestrained pull of the ulnar finger flexors.

A 47-year-old male with a right MCA stroke status post hemicraniectomy with left hemiparesis and right gaze preference has become aggressive with therapies over the past two days. What is the appropriate next step? A.Start a mood stabilizing agent B.Check a lumbar puncture C.Check serum chemistries D.Modify environmental factors

C. Check serum chemistries 47.24%Electrolyte abnormalities and infections need to be ruled out in this patient with a change in mental status. A lumbar puncture is not the appropriate next step. Modification of environmental factors or the initiation of a mood stabilizing agent should not be considered until a physiologic cause has been eliminated.

A 60-year-old male is noted to have difficulty walking in the rehabilitation gym after a left total knee arthroplasty five days earlier. On examination, he has 70o of active knee flexion and a 20o extensor lag. His distal limb is normal to palpation. You notice that he has trouble clearing his toes during the swing phase of gait. You suspect the major cause of his difficulty walking is due to: A.Weak quadriceps muscle strength B.Inadequate knee flexion range C.Common fibular nerve palsy D.Tibialis anterior tendon tear

C. Common fibular nerve palsy 55.02% The patient has a common fibular (peroneal) nerve palsy which can occur after total knee arthroplasty. Weak quadriceps muscle strength and inadequate knee flexion may cause difficulty with ambulation, but should not cause loss of ankle dorsiflexion. A tibialis anterior tendon tear will cause difficulty with ankle dorsiflexion, but is not a common complication after knee arthroplasty. Also, acute tendon tears present with sudden pain and palpatory defect.

A 55-year-old female presents to the clinic with a 6-week history of right wrist pain. She has been taking anti-inflammatory medications without relief. You diagnose her with de Quervain tenosynovitis. What is the next most appropriate step in treatment? A.Trial of a higher dose of anti-inflammatory medication B.Iontophoresis of the affected area C.Corticosteroid injection D.Surgical consultation

C. Corticosteroid injection 95.44% Corticosteroid injection for de Quervain tenosynovitis has been shown to be more effective treatment than iontophoresis and anti-inflammatory medications.

A 60-year-old male presents with medial elbow pain that radiates to the medial hand. The pain is aggravated by prolonged elbow flexion. On examination, Tinel sign at the medial elbow is positive. What is the patient's diagnosis? A.Carpal tunnel syndrome B.Tarsal tunnel syndrome C.Cubital tunnel syndrome D.Guyon's canal syndrome

C. Cubital tunnel syndrome 98.48% Cubital tunnel syndrome is a type of ulnar neuropathy at the elbow due to entrapment or compression of the ulnar nerve under the humeroulnar aponeurotic arcade. Tarsal tunnel syndrome is compression of the posterior tibial nerve under the flexor retinaculum of the medial ankle. Carpal tunnel syndrome is compression of the median nerve at the wrist under the transverse carpal ligament. Guyon's canal syndrome is compression of the ulnar nerve at the wrist, under the palmar carpal ligament.

Critical illness polyneuropathy and myopathy are often seen in the intensive care setting and accompanied by: A.Acute inflammatory demyelinating polyneuropathy B.Post-procedure complications after spinal decompressive surgery C.Difficulty weaning from mechanical ventilation D.Myasthenia-Gravis Syndrome

C. Difficulty weaning from mechanical ventilation 91.79% Failure to wean from mechanical ventilation is often the presenting reason for electrodiagnostic medicine consultation. Critical illness polyneuropathy is the most common severe neuropathy seen in the ICU. Critical illness polyneuropathy and myopathy are most commonly due to a medical illness, such as sepsis or multi-organ failure, and not attributed to other comorbid neuromuscular disorders (such as AIDP or Myasthenia Gravis Syndrome.)

What is the most sensitive neuroimaging technique to detect diffuse axonal injury after traumatic brain injury? A.T1-weighted MRI scan B.T2-weighted MRI scan C.Diffusion-weighted MRI scan D.FLAIR sequence MRI

C. Diffusion-weighted MRI scan 70.69% Standard computed tomography and magnetic resonance imaging (MRI) scans are often unremarkable after isolated diffuse axonal injury (DAI) in patients with significant cognitive and behavioral problems. However, fluid-attenuated inversion recovery (FLAIR), diffusion-weighted and gradient echo MRI sequences are more sensitive in detecting evidence of DAI after a TBI. The T2 weighted image is more sensitive to intraparenchymal blood than is FLAIR and can reveal hemorrhagic DAI, it also provides greater detail on brainstem and central gray matter.

Which of the following is a typical feature of seronegative spondyloarthropathies? A.Symmetric arthritis B.Bursitis C.Enthesitis D.Vasculitis

C. Enthesitis 81.72% Seronegative spondyloarthropathies (SpA) are commonly asymmetric and have familial patterns. Less than 5% of patients are HLA-B27 negative. In order to identify patients with SpAs, symptoms of LBP lasting ≥ 3months should be present, and occur before age 45 years. In addition, the following criteria should be met: Sacroiliitis on imaging plus one or more of the SpA features listed below. Or, alternatively, HLA-B27 positivity plus two or more of the following SpA features listed: inflammatory back pain; arthritis; enthesitis (e.g., heel); uveitis; dactylitis; Psoriasis; Crohn's/colitis; good response to NSAIDs; family history of SpA; elevated C-reactive protein (CRP).

A 25-year-old male reporting pain about the posterior aspect of the right shoulder is found to have a hypoechoic region on ultrasonography along the origin of the deltoid muscle. There is also accompanying vascular flow on power Doppler imaging. His review of symptoms is positive for frequent loose stools and abdominal pain and bloating. What should you suspect? A.Muscle strain B.Impingement C.Enthesitis D.Vascular malformation

C. Enthesitis 82.41% On sonographic evaluation, enthesitis is characterized by regions of hypoechogenicity in association with blood flow seen on power Doppler imaging. Of note, enthesitis along the origin of the deltoid at the acromion may be associated with spondyloarthropathies.

Minimally conscious state is defined by the presence of: A.Sleep-wake cycles B.Cranial nerve reflexes C.Environmental awareness D.Reflexive behaviors

C. Environmental awareness 68.69% Minimally conscious state refers to the return of self- or environmental awareness. The vegetative state is characterized by the presence of sleep-wake cycles. Cranial nerve reflexes may be preserved in the vegetative state. One must be careful that reflexive behaviors such as yawning or auditory startle not be considered true awareness.

A 42-year-old female with a history of rheumatoid arthritis (RA) presents to the office with complaints of chronic neck pain. Which of the following radiological findings should prompt referral to a spine surgeon? A.Stability of the atlantoaxial (AA) articulation on lateral flexion/extension radiographs B.Disc space narrowing with associated endplate sclerosis on lateral radiographs C.Erosion of the odontoid on sagittal MRI scan D.Schmorl's nodes noted on sagittal MRI scan

C. Erosion of the odontoid on sagittal MRI scan 86.73% Erosion of the odontoid and AA instability is a classic presentation of rheumatoid arthritis affecting the cervical spine. A distance greater than 2.5 mm between the inferior aspect of the posterior arch of C1 and the anterior dens is diagnostic of instability typically from disruption of the transverse ligament by pannus formation. Axial migration of C2 is not uncommon as the odontoid erodes and a potential space is created. Any of these radiological findings in a patient with RA (even in the presence of a normal neurological examination) should prompt referral to a spine surgeon for consideration of an occipital-cervical fusion as these patients are at heightened risk for sudden death secondary to brainstem or spinal cord compression. Disc space narrowing with associated sclerosis and Schmorl's nodes in the absence of any of the other radiological findings would not necessitate surgical spinal consultation.

What is the most common primary malignant tumor of the brain in adults? A.Medulloblastoma B.Meningioma C.Glioblastoma multiforme D.Ependymoblastoma

C. Glioblastoma multiforme 86.63% More than 90% of the primary malignant tumors of the brain in adults are high-grade astrocytomas and, of these, the most common is glioblastoma multiforme. Meningiomas are tumors that occur in the membranes that cover and protect the brain and spinal cord (the meninges). Meningiomas usually grow slowly. Medulloblastomas are almost always found in children or young adults. Ependymoblastomas are rare cancers that usually occur in children.

A 5-year-old with spastic diplegic cerebral palsy presents with asymmetric gait, positive Galeazzi sign with limb shortening, decreased hip abduction and decreased knee extension on the left compared to the right. What is the cause of these abnormalities? A.Worsening scoliosis B.Hamstring tightness C.Hip subluxation D.Ankle contracture

C. Hip subluxation 75.38% The Galeazzi sign refers to the comparison of knee height. Asymmetric knee heights with the hips flexed imply hip subluxation on the apparently shorter side. Hip subluxation is common in cerebral palsy and is often suspected on examination when there is decreased abduction and shorter knee height. Diagnosis is made with radiographs in children and by ultrasound in infants under the age of 6 months.

A patient with multiple sclerosis undergoes an eye exam that reveals the following: a) pupils are round and reactive to light; b) upon rightward gaze, left eye does not cross midline; c) right eye demonstrates horizontal nystagmus; d) diplopia when looking to the right but not straight ahead or to the left. These findings are consistent with: A.Parinaud syndrome B.Horner syndrome C.Internuclear ophthalmoplegia D.Millard-Gubler syndrome

C. Internuclear ophthalmoplegia 95.14% Internuclear ophthalmoplegia (INO) is caused by a lesion in the medial longitudinal fasciculus (MLF) in the paramedian brainstem, rostral to the abducent nucleus. It is characterized by impaired adduction of the eye ipsilateral to the lesion with gaze toward the contralateral side of the lesion. In practice, an isolated case of INO is rare since the two sides of the MLF are very near the midline of the brain stem. The two most common causes of INO are multiple sclerosis and paramedian brain stem infarct. The Horner syndrome is due to a superior cervical sympathetic ganglion lesion and causes miosis, ptosis and anhydrosis. Parinaud syndrome causes impaired upward gaze with dilated and nonreactive pupils and is the result of a lesion in the midbrain, usually a pineal tumor. Millard-Gubler syndrome is due to an ipsilateral pons lesion causing ipsilateral palsy of cranial nerve (CN) 6 and CN 7 and a contralateral hemiparesis and tactile sensory loss.

Which right-sided radiculopathy level is consistent with the needle EMG abnormalities depicted below? (Editorial: Abn Spont activity in right rectus femoris, Adductor longus, and TFL; Normal Iliacus, vastus medialis, tib ant, gastroc, EHL, Abd Hallucis) -TG A.L2 B.L3 C.L4 D.L5

C. L4 58.05% Iliopsoas and all the quadriceps (vastus medialis, vastus lateralis, vastus intermedius, and rectus femoris) are innervated by the femoral nerve, and to a varying degree, are supplied by L2, L3, and/or L4 roots; needle EMG could demonstrate abnormalities in these muscles in L2, L3, or L4 radiculopathy. The tensor fascia lata is innervated by the superior gluteal nerve, and is predominantly supplied by the L4 and L5 roots. The only myotomal level shared by the abnormal muscles is L4.Although nerve conduction studies would aid in the diagnosis of femoral neuropathy, the involvement of the tensor fascia lata would suggest otherwise. A poorly-placed needle, however, intended for the tensor fascia lata could inadvertently record from within the rectus femoris, misleading the practitioner.

A 66-year-old woman with a history of rheumatoid arthritis (RA) presents with a 3-week history of left ankle and calf pain with difficulty walking. She reports that her RA affects her hands, wrists, and ankles the most. Two weeks ago, she fell while walking at home and has been using a manual wheelchair since then. On exam, both ankles have a mild effusion and her left calf is tender to palpation. She has a positive Thompson test on the left, but not on the right. What test/procedure would be most diagnostic? A.Joint aspiration B.Plain radiograph C.Magnetic resonance imaging D.Venous ultrasound

C. Magnetic resonance imaging 70.23% The patient has a left Achilles tendon tear likely secondary to tenosynovitis from her rheumatoid arthritis (RA). Most tendon ruptures in RA are abrupt. Her examination is characteristic of an acute Achilles tendon rupture with lack of plantar flexion on calf squeeze (ie, a positive Thompson test). An MRI scan of the patient's left ankle will most readily demonstrate interstitial tearing of the tendon. Joint aspiration, plain radiograph, and venous ultrasound would not be diagnostic.

Which measure often used to identify the existence of comorbid psychological problems present in chronic pain patients and to help tailor treatment? A.Fear-avoidance beliefs questionnaire (FABQ) B.Oswestry Disability Index (ODI) questionnaire C.Minnesota multiphasic personality inventory (MMPI) D.Short Form 36 (SF-36)

C. Minnesota multiphasic personality inventory (MMPI) 36.21% The MMPI and its variants (version 2 and 2RF) are the most widely used psychometric tests for adult personality and psychopathology testing.

Which of the following needle EMG findings is more characteristic for radiation plexopathy compared to plexopathy due to tumor involvement? A.Positive sharp waves B.Fasciculations C.Myokymia D.Myotonia

C. Myokymia 96.35% Myokymia is seen in 60% of patients who received radiotherapy for treatment of cancer for (e.g. breast and lung). Radiation plexopathy can be seen from 3 months to 14 years with median of 1.5 years. Myokymia is less frequently seen in patients with tumor plexopathy.

What cervical spine imaging study for atlantoaxial instability (AAI) should be included in a sports participation physical for an asymptomatic child with Down syndrome? A.Ultrasound examination B.Magnetic resonance imaging C.Neutral, flexion and extension x-rays D.Computed tomography imaging

C. Neutral, flexion and extension x-rays 93.62% Initial screening neutral, flexion and extension cervical spine films are recommended in the United States to assess for AAI since it is common (13-14%) in Down syndrome. An anterior atlantodens interval (ADI) of >3.5 mm represents instability and recommendation to avoid sports with potential neck flexion/extension injuries is warranted. Repeating films yearly is not necessary as stability over time has been demonstrated. CT or MRI are not needed as screening tools but are utilized to work up a symptomatic individual.

What does the term maximum medical improvement (MMI) mean? A.A patient's condition is no longer clinically significant. B.No further treatment for the condition is required. C.No further treatment is reasonably expected to improve the condition. D.The physician and patient agree that the condition has stabilized.

C. No further treatment is reasonably expected to improve the condition. 95.44% Maximum medical improvement (MMI) is a term used to indicate that further significant recovery or deterioration of a condition is not anticipated to occur. The patient's condition may remain clinically symptomatic, as resolution of the condition may or may not occur. Ongoing treatment (eg, maintenance treatment) may be required following MMI.

An infant presents with subdural hematoma, retinal hemorrhages, and rib and femur fractures. The most likely etiology is: A.Von Willebrand disease B.Osteogenesis imperfecta C.Non-accidental trauma D.Hemophilia

C. Non-accidental trauma 94.83% Children with hemophilia and von Willebrand disease may have bruising or bleeding disproportionate to the reported trauma. Children with osteogenesis imperfecta and those with metabolic bone disease can have fractures of varying ages that are out of proportion to the reported trauma. Red flags for non-accidental trauma include injuries out of proportion to developmental ability (i.e., femur fracture when unable to walk), posterior rib fractures, retinal hemorrhages, and subdural hematoma.

A 53-year-old woman was treated for Hodgkin lymphoma with 3600 cGy of mantle field radiation when she was 23 years old. She now presents to your clinic with progressive difficulty holding her head erect and upper extremity weakness. Which of the following is LEAST likely to be contributing to her symptoms? A.Radiculopathy B.Plexopathy C.Polyneuropathy D.Myopathy

C. Polyneuropathy 20.39% Mantle field radiation includes all the lymph nodes in the neck, chest, and axilla. All structures in the radiation field are subject to damage, which can become clinically evident years later and can progress indefinitely. In addition to viscera such as the heart and lungs, mantle field radiation can damage all neuromuscular structures in the field including the spinal cord, nerve roots, plexus, local nerves, and muscles. This has been termed a "myelo-radiculo-plexo-neuro-myopathy." The term "polyneuropathy" describes a diffuse neuropathic process affecting peripheral nerves. The damage to named and unnamed peripheral nerves from focus radiation is confined to the radiation field (i.e., multiple mononeuropathies) and therefore this term is not appropriate.

In which of the following conditions would a unilateral absent tibial H reflex most likely be seen? A.Polyneuropathy B.Lumbar plexopathy C.S1 Radiculopathy D.Myelopathy

C. S1 Radiculopathy 95.52% In polyneuropathy, the H reflex may be delayed or absent bilaterally. Upper motor neuron pathology such as myelopathy alone would not affect the H reflex. Lumbar plexopathy is not associated with H reflex abnormalities, though a lumbosacral plexopathy could demonstrate abnormalities. A unilaterally absent H reflex is most indicative of S1 radiculopathy.

A 50-year-old man with a 6-week history of right leg weakness is sent for electrodiagnostic studies. On examination he has weakness with right ankle dorsiflexion, eversion, and great toe extension. Sensation is decreased over the dorsum of the foot. Electrodiagnostic studies show the following. (Editorial: NCS: diffusely low amplitude R peroneal motor - no drop across the fib head, normal latency; absent R superficial peroneal sensory; slight side to side R<L tibial motor amplitude; normal surals. EMG: Abn spont activity and red recr in R semimebranosus, SH biceps, Tib ant, medial gastroc, Peroneus longus, tib posterior; Normal paraspinals, Glut Max, vastus, adductor longus, TFL) -TG The patient's symptoms are most likely due to right: A.Fibular nerve injury at the fibular head B.Deep fibular nerve injury C.Sciatic nerve injury D.Lumbosacral plexopathy

C. Sciatic nerve injury 77.81% The lesion is most consistent with a sciatic nerve mononeuropathy affecting the fibular (peroneal) fibers more profoundly.Muscles innervated by both the fibular (peroneal) and tibial divisions show abnormalities on needle examination. The lesion is distal to the origin of the gluteal nerves from the lumbar plexus, and muscles supplied by the femoral and obturator nerves are unaffected. The right motor and sensory responses from the foot are smaller or absent compared to the left side. The smaller sensory responses and normal needle examination of the lumbosacral paraspinal muscles makes a root lesion less likely. The clinical presentation is suggestive of just a fibular (peroneal) nerve injury - this is not unusual in cases of sciatic nerve lesions where the tibial portion is less involved.

According to the Americans with Disabilities Act (ADA), an individual with a T12 AIS A spinal cord injury would likely be able to perform the essential job functions of which of the following positions without accommodation? A.Truck driver B.Plumber C.Software developer D.Warehouse worker

C. Software developer 98.97% Accommodation would be needed for someone with paraplegia to work as a commercial truck driver. The individual presumably has no training as a plumber and would therefore be unable to work as plumber. An analyst is a sedentary clerical position that could be done by a person with paraplegia with readily available computer interface approaches. A warehouse clerk is a physically demanding job that would likely require some type of accommodation for a person with paraplegia (e.g., a robotic exoskeleton), if any were available.

A 1-month-old boy exhibits generalized hypotonia and weakness. Which of the following should be included in the differential diagnosis? A.Lumbosacral myelomeningocele B.Duchenne muscular dystrophy C.Spinal muscular atrophy D.Neuroblastoma

C. Spinal muscular atrophy 92.4% Myelomeningocele usually involves weakness of the legs, but not generalized hypotonia. Duchenne muscular dystrophy usually presents after age 2 years with weakness, but attainment of motor milestones is usually normal to mildly delayed. Neuroblastoma may present with leg paralysis, but not generalized hypotonia. The differential diagnosis of generalized hypotonia and weakness in an infant should include brain pathology (eg, severe encephalopathy), proximal spinal cord disorders (eg, spinal cord injury or spinal muscular atrophy), and myopathy.

Which salivary glands should be targeted with botulinum toxin to control anterior drooling when there is no food stimulation? A.Sublingual B.Parotid C.Submandibular D.Buccal

C. Submandibular 41.03% Targeting the appropriate salivary glands is essential. Since the submandibular glands are the major producers of saliva when the patient is not stimulated by food, and parotid glands produce mainly during feeding or other oral motor stimulation, these are the glands to target. Because the sublingual glands produce less than 5% of saliva, they typically are not injected. The buccal glands are located in the mucous membrane lining the cheeks and mouth. These glands produce only a small amount of saliva. The parotid glands can be localized by surface anatomy, whereas the submandibular glands may require ultrasound guidance. Current studies show significantly decreased anterior drooling after botulinum toxin A injections to the submandibular and parotid glands. It should be noted that use of botulinum toxin A (Botox) is off-label for those under 18 years of age.

A 54-year-old male with a prior history of a right L5 lumbar radiculopathy from a work injury status-post lumbar fusion presents with recurrent radicular symptoms in the same distribution. Which of the following is correct regarding apportionment of his condition? A.Documentation of a prior injury is not necessary B.Apportionment is not indicated if the previous injury occurred with a different employer C.There is evidence (> 50% likelihood) the prior injury contributed to the current impairment D.Calculation of apportionment is the same from state to state

C. There is evidence (> 50% likelihood) the prior injury contributed to the current impairment 82.98% When apportioning a condition, the provider must show that a prior condition or injury has contributed to the current impairment. Apportionment also requires documentation of a prior condition or injury. Employment status is not a factor in determining apportionment. Calculation of apportionment may vary from state to state.

A 60-year-old patient complains of pain and tenderness over the lateral right hip. She is unable to sleep on that side. Physical examination notes tenderness over the lateral greater trochanter. What is the most likely diagnosis? A.Osteoarthritis of the hip B.Septic arthritis C.Trochanteric bursitis D.Trochanteric tendonitis

C. Trochanteric bursitis 91.38% The patient has symptoms of trochanteric bursitis. Osteoarthritis of the hip would be painful with internal rotation and there would be evidence on radiographic imaging. Septic arthritis would be associated with fever and severe pain with motion. Trochanteric tendonitis would have tenderness proximal to the trochanter suggesting tendinitis of the gluteus medius tendon.

In patients with a history of ventricular arrhythmias and ischemic heart disease, exercise stress tests are: A.Performed in the supine position to decrease myocardial oxygen demand B.Avoided due to the high risk of ventricular arrhythmias during testing C.Used to determine a safe target heart rate that does not provoke arrhythmias D.Not necessary in patients with good premorbid exercise tolerance

C. Used to determine a safe target heart rate that does not provoke arrhythmias 53.19% Exercise stress tests are used to screen for ventricular arrhythmias and to determine the target heart rate, which is set below the level at which arrhythmias are noted. Upright exercise produces less myocardial oxygen demand than supine exercise and therefore patients prone to arrhythmias should be advised to exercise in the upright position. Approximately 80% of patients with a history of ventricular arrhythmia will have a ventricular arrhythmia during inpatient cardiac rehabilitation.

A 22-year-old woman with complaints of fluctuating weakness and abnormal fatigability that improves with rest is sent for electrodiagnostic testing. Which electrodiagnostic finding on routine testing would be most consistent with this clinical presentation? A.Decreased motor unit recruitment B.Slow motor nerve conduction velocities C.Variability in motor unit action potential amplitude D.Small compound muscle action potentials

C. Variability in motor unit action potential amplitude 78.42% This patient's presentation is most consistent with myasthenia gravis. The incidence of this condition is bimodal and affects women more than men in the younger age group. When myasthenia gravis is suspected, the test of choice is repetitive nerve stimulation. However, it is still important to assess for other possible problems and routine nerve conduction and needle electromyographic examination should be performed. The sensory component of the peripheral nervous system lacks a neuromuscular junction and hence the sensory responses should be normal. Motor amplitudes can be small, but this is usually only in severe cases. Motor conduction velocities are normal, since this study assesses the conduction along the motor fibers. Motor unit action potential amplitude variability is a characteristic abnormality observed during routine electromyography in patients with neuromuscular junction disorders. This finding is due to the variability in the total number of single muscle fibers being activated at any single time.

A 65-year-old file clerk presents with low back pain that occurred 6 months ago after he bent over to pick up a file at work. He saw his primary care physician, who ordered magnetic resonance imaging (MRI) of his lumbar spine and told him he had a bulging disc. He was subsequently referred to you because he is still symptomatic. The patient feels his bulging disc was caused by bending over to pick up the file at work. You tell him that his bulging disc: A.Is uncommon for a 65-year-old individual B.Occurred when he bent over to pick up the file C.Was likely present before the onset of his low back pain D.Is pinching a nerve and causing his low back pain

C. Was likely present before the onset of his low back pain 89.36% Bulging discs are seen commonly in 65-year-old individuals. In a study of asymptomatic individuals, 79% of those age 60 or older had bulging discs. Other studies have confirmed the finding that bulging discs occur more commonly in older individuals.

A patient with multiple myeloma presents with a new pathologic fracture of the right superior pubic ramus. Computed tomography scans and plain radiographs fail to reveal additional myelomatous involvement of the pelvic ring. When consulted regarding weight-bearing recommendations you advise: A.Complete bed rest for six weeks B.Non-weight bearing on right lower extremity C.Weight bearing as tolerated D.Toe touch weight bearing until surgical stabilization

C. Weight bearing as tolerated 53.19% Pathologic fractures of the pelvis that do not involve the acetabulum are generally treated non-surgically. Patients may bear weight as tolerated. Aggressive analgesic management may be required. Mechanical insufficiency of the acetabulum can only be managed surgically with reconstruction using screws or pins with protrusioacetabular component.

A 10-year-old track and field athlete presents with tenderness at the inferior pole of the patella. What is the likely diagnosis? A.Patellar tendinopathy B.Osgood-Schlatter disease C.Sinding-Larsen-Johansson disease D.Pes anserine bursitis

C. Sinding-Larsen-Johansson disease 36.78% The tenderness at the inferior pole of the patella is suggestive of Sinding-Larsen-Johansson disease. Patellar tendinopathy will have pain at the patellar tendon, and Osgood-Schlatter disease at the tibial tuberosity. Pes anserine bursitis will have pain at the proximal antero-medial tibia, and quadriceps tendinopathy will have pain at the quadriceps tendon, usually at the insertion of the superior pole of the patella.

The correct key sensory point for L3 according to the international standards for neurological classification of SCI, is the: A.Superior edge of the patella B.Lateral femoral condyle C.Medial femoral condyle D.Midline of tibial plateau

C.Medial femoral condyle82.85% L3: medial femoral condyle above the knee. None of the other points correlate to a key sensory point.

A 28-year-old man returns to clinic after failing conservative management for lateral epicondylosis. What is the most cost-effective diagnostic test to localize the site of pathology? A.Plain radiographs B.Magnetic resonance imaging C.Real time ultrasound D.Electrodiagnostic studies

C.Real time ultrasound 89.06% Real time ultrasound is less costly than magnetic resonance imaging (MRI) and has similar sensitivity and specificity in diagnosing lateral epicondylosis. Plain radiographs and electrodiagnostic studies will not help localize or confirm your diagnosis of lateral epicondylosis, but may help with diagnosing a fracture or nerve injury, respectively.

A 30-year-old throwing athlete presents with a history of sudden onset right shoulder pain one month ago, followed by progressive weakness in the same arm. Motor conduction studies are normal for both median and ulnar nerves. Sensory conduction studies are absent in lateral cutaneous nerve of the forearm (lateral ante-brachial cutaneous) median to first digit and radial to first digit. EMG demonstrates fibrillations, positive sharp waves and reduced recruitment in supraspinatus, deltoid, biceps brachii, brachioradialis, pronator teres, rhomboid, serratus anterior, triceps and extensor carpi radialis longus. Flexor pollicis longus and first dorsal interosseous and cervical paraspinals are normal. What is the most likely diagnosis? A.C5 radiculopathy B.C6 radiculopathy C.Upper trunk plexopathy D.Lateral cord plexopathy

C.Upper trunk plexopathy 80.85% The muscle domains of upper brachial plexus include serratus anterior, rhomboids supraspinatus infraspinatus, biceps brachii, deltoid, teres minor, triceps pronator teres, flexor carpi radials, brachioradialis, and extensor carpi radialis. Flexor pollicis longus and first dorsal interosseous are C8 T1 lower plexus medial cord muscles.C5-C6 radiculopathy is unlikely since sensory responses are absent from lateral cutaneous nerve of the forearm (lateral ante brachial cutaneous), median and radial to first finger as well as negative paraspinals. Deltoid, suprascapular nerve innervated supraspinatus and dorsal scapular nerve innervated rhomboid places the lesion proximal to the level of lateral cord. Even though triceps and brachioradialis are affected, this lesion is more rostral than posterior cord since muscles as serratus anterior rhomboid are involved. Pure posterior cord lesions should not involve biceps and pronator teres muscles. Therefore, this is an upper brachial plexus lesion at the trunk level.

Repeatedly lifting the shoulder past which degree of flexion or abduction is associated with an increased prevalence of shoulder disorders? A.10° B.30° C.45° D.60°

D. 60° 78.12% Repeatedly lifting the shoulder past 60 degrees of flexion or abduction is associated with an increased prevalence of shoulder disorders.

A distance of 13.5 cm on the Shober test during evaluation of a 25-year-old male with low back pain would be consistent with: A.Lumbar spondylolisthesis B.Scheuermann disease C.Lumbar herniated disc D.Ankylosing spondylitis

D. Ankylosing spondylitis 88.45% The Schober test is used to assess restricted range of motion seen in ankylosing spondylitis as the disease progresses. The distance between the 2 points with forward flexion should exceed 15cm in non-affected individuals.

A college basketball player present with knee pain and swelling after he suffered an injury to his knee while coming down from a rebound with an awkward landing. On initial radiographs, you find an avulsion fracture off the lateral tibial plateau. What structure is most likely to be injured? A.Lateral collateral ligament B.Posterior meniscofemoral ligament C.Posterior cruciate ligament D.Anterior cruciate ligament

D. Anterior cruciate ligament 42.55% The above finding on x-ray is a Segond fracture, which is considered pathognomonic for the presence of an anterior cruciate ligament (ACL) tear.

Which of the following is good practice to prevent electrical injury during electrodiagnostic studies? A.Use a minimum of two electrodes during nerve conduction studies B.Turn machines on before attaching electrodes to the patient and turn them off before disconnecting the patient C.Use a stimulus pulse duration of 0.2 ms or greater D.Avoid extension cords

D. Avoid extension cords 67.59% Leakage current increases with the length of the power cord, therefore extension cords should be avoided. A is incorrect because ground electrodes should always be used to dissipate leakage current, therefore nerve conduction studies are performed with a reference electrode, and active electrode, and a ground electrode. The machine should be turned on before attaching electrodes to the patient and after disconnecting them. Using a stimulus pulse duration greater than 0.2ms in someone with an implanted pacemaker or cardioverter increases the risk that the stimulus may be misinterpreted as a QRS complex.

A 65-year-old woman complains of chronic back pain for "decades" and increased difficulty with movement. There is no history of recent trauma. Radiographs of her thoracolumbar spine are taken and demonstrate relative preservation of disc height along with continuous calcifications along the anterolateral areas. The sacroiliac joints are unremarkable. What is her diagnosis? A.Ankylosing spondylitis B.Spondylosis deformans C.Intervertebral osteochondrosis D.Diffuse idiopathic skeletal hyperostosis

D. Diffuse idiopathic skeletal hyperostosis 85.11%Radiographs of diffuse idiopathic skeletal hyperostosis (DISH) demonstrate calcification of the anterior longitudinal ligament. Criteria for DISH include the relative preservation of intervertebral disk height, flowing anterolateral calcifications of at least 4 continuous vertebral levels, and the absence of sacroiliac joint erosions. It differs from ankylosing spondylitis by the lack of involvement of the sacroiliac joint (erosions, sclerosis). Spondylosis deformans results in the formation of large osteophytes along the vertebral bodies which are typically right-sided with initial horizontal orientation. Intervertebral osteochondrosis is a primary degenerative disease of the nucleus pulposus.

Which of the following is a cause of non-erosive arthritis? A.Rheumatoid Arthritis B.Gout C.Psoriasis D.Lupus

D. Lupus 63.53% Rheumatoid arthritis, gout, and psoriasis are all articular erosive arthritides. Lupus does not typically cause erosive arthritis.

A patent presents with complaints of hand weakness and numbness and tingling along lateral hand, with atrophy of his hand interossei muscles. Which of the following is the BEST muscle to study with needle EMG to distinguish an ulnar mononeuropathy from a C8 or T1 radiculopathy? A.First dorsal interosseous B.Flexor carpi ulnaris C.Flexor carpi radialis D.Pronator quadratus

D. Pronator quadratus 47.9% While the first dorsal interosseous muscle is innervated by the ulnar nerve, the pronator quadratus has C8 innervation from the median nerve via the anterior interosseous branch. The flexor carpi radialis has median nerve innervation, but from roots C6-C7.

A 45-year-old patient with a history of hyperparathyroidism presents with recurrent episodes of knee pain. X-rays reveal chondrocalcinosis of the medial and lateral menisci. The most likely diagnosis which would account for the patient's pain and x-ray findings is: A.Osteoarthritis B.Gout C.Paget's disease D.Pseudogout

D. Pseudogout 76.9% A definitive diagnosis of CPPD arthropathy (ie, pseudogout) requires the identification of CPPD crystals from joint fluid; however, the radiologic findings in this case are diagnostic. CPPD deposition disease can be associated with hyperparathyroidism, hemochromatosis, and amyloidosis. It is weakly associated with hypothyroidism. Chondrocalcinosis is not seen in osteoarthritis, monosodium urate crystal arthropathy (ie, gout), or Paget's disease.

Abnormal needle EMG may be seen in which of the following muscles in the setting of femoral mononeuropathy? A.Gracilis muscle B.Adductor magnus muscle C.Adductor longus muscle D.Sartorius muscle

D. Sartorius muscle 69.66% The sartorius muscle is innervated by the femoral nerve. The gracilis and adductor longus muscles are innervated by the obturator nerve. The adductor magnus muscle has dual innervation from the obturator nerves and the tibial portion of the sciatic nerve.

What laboratory test demonstrates high yield in the workup of acquired (nonhereditary) distal symmetric polyneuropathy? A.Antinuclear antibody (ANA) B.Cytoplasmic antineutrophil cytoplasmic antibodies (c-ANCA) C.Liver function tests D.Serum protein immunofixation electrophoresis

D. Serum protein immunofixation electrophoresis 66.02% Screening laboratory tests may be considered for all patients with distal symmetric polyneuropathy (DSP). Although routine screening with a panel of basic tests is often performed, those tests with the highest yield of abnormality are blood glucose, serum B12 with metabolites (methymalonic acid with or without homocysteine), and serum protein immunofixation electrophoresis. When routine blood glucose testing is not clearly abnormal, other tests for pre-diabetes (impaired glucose tolerance) such as a glucose tolerance test may be considered in patients with distal symmetric sensory polyneuropathy, especially if it is accompanied by pain.Although there are no controlled studies regarding when to recommend the use of other specific laboratory tests, clinical judgment correlated with the clinical picture will determine which additional laboratory investigations are necessary.

Which of the following is commonly used as an outcome measure to assess perception of physical and emotional functioning in patients with chronic pain? A.Fear-avoidance beliefs questionnaire (FABQ) B.Oswestry Disability Index (ODI) questionnaire C.Minnesota multiphasic personality inventory (MMPI) D.Short Form 36 (SF-36)

D. Short Form 36 (SF-36) 18.54% The SF-36® and the shorter SF-12® is a measure of quality of life with questions covering multiple domains. SF-36 Domains include "physical functioning; role limitations due to physical health; role limitations due to emotional problems; energy/fatigue; emotional well-being; social functioning; pain; general health". These questionnaires are owned and marketed by Medical Outcomes Trust. They have a considerable research base and assess a broad range of activities of daily living relevant to disability.

Which electroencephalogram pattern is associated with a better prognosis after traumatic brain injury? A.Low amplitude delta activity B.Burst suppression C.Isoelectric activity D.Spindle pattern

D. Spindle pattern 37.86% Favorable electroencephalogram (EEG) patterns after a traumatic brain injury are normal activity, rhythmic theta activity, frontal rhythmic delta activity, and spindle pattern. Poor prognosis is associated with epileptiform activity, nonreactive, low amplitude delta activity and burst suppression patterns with interruption of isoelectricity. Complete isoelectric EEG activity had the highest mortality.

A 23-year-old student recently started training for a half marathon, but has been limited by bilateral knee pain. She reports anterior knee pain and describes it as "beneath the knee cap." The pain is worse when arising after prolonged sitting position. Which physical examination finding might be expected? A.Pes cavus B.Weak hip adductors C.Flexible iliotibial band D.Tight quadriceps muscles

D. Tight quadriceps muscles 79.64% Patellofemoral arthralgia is thought to result from tracking problems of the patella within the trochlear groove. Several biomechanical issues, such as tight and inflexible quadriceps, pes planus or over pronation, tight iliotibial band, weak and ineffective vastalis medialis, and weak hip abductors may contribute to incorrect tracking of the patella. The Ober test assesses the tensor fascia lata and iliotibial band for contracture and inflexibility.

Which of the following measures of the extremity arterial blood supply is predictive of a favorable outcome in the healing of a foot sore? A.Toe blood pressure of 20mmHg B.Absent dorsalis pedis and posterior tibial pulses C.Ankle-brachial index less than 0.45 D.Transcutaneous oxygen tension greater than 35mmHg

D. Transcutaneous oxygen tension greater than 35mmHg 80.55% Various methods for the vascular assessment of the lower extremity are available for predicting healing of foot sores. The ankle-brachial index > 0.45 is the cornerstone of non-invasive vascular testing; however, the transcutaneous oxygen saturation greater than 35mmHg is a more specific indicator of tissue perfusion and viability. Absolute toe blood pressures and distal foot pulses are also used for screening. Absent distal pulses and low absolute toe blood pressures are poor prognostic indicators.

Which of the following signifies progression from a vegetative state to a minimally conscious state? A.Spontaneous eye opening B.Sleep wake cycle C.Spontaneous breathing D.Visual tracking

D. Visual tracking 77.02% Spontaneous eye opening and presence of sleep wake cycle signifies the vegetative state. The diagnosis of MCS is based on clearly discernible and reproducible evidence of purposeful movements.

A 35-year-old woman presents with a 3-4 week history of numbness in the right little finger and ulnar aspect of the right hand and weakness with handgrip strength compared to the other side. Her electrodiagnostic studies show the following. Figures show: -Normal sensory NCV -Motor NCV with slight side-to-side amplitude difference (R<L) with median and ulnar studies but normal CV and latencies. -Needle study shows reduced recruitment and abnormal spontaneous activity in EDC, PQ, ECU, FDI, APB, and cervical paraspinals on the right. These findings are most consistent with right: A.Ulnar and median mononeuropathies B.Lower trunk plexopathy C.Ulnar mononeuropathy D.C8/T1 radiculopathy

D.C8/T1 radiculopathy 83.82% The electromyographic findings are localized to muscles supplied by the C8/T1 roots. The triceps can have some C8 innervation but frequently tests normal in C8/T1 radiculopathies. The normal sensory studies make it less likely that this is a plexopathy or peripheral nerve injury. The median and ulnar motor amplitudes are in the normal range but smaller compared to the left side. Finally, the involvement of the lower cervical paraspinal muscles is consistent with a cervical radiculopathy.

An 80-year-old woman with a history of dementia diagnosed three years ago is brought to your clinic by her family. The patient had been gradually deteriorating but has been coping at home with family support. She is confused at baseline. Two days ago, she became incontinent of urine (unusual for her), more confused with occasional return to baseline cognition, disorganized and inattentive in her conversation with family. The family has noted increased lethargy. Balance has remained unchanged from baseline. Her change in symptoms is most consistent with: A.Dementia B.Normal pressure hydrocephalus C.Depression D.Delirium

D.Delirium 67.78% As this patient has dementia, she is likely to be at a particularly high risk of delirium. Worsening dementia is likely, due to the long history and gradual deterioration, but not due to the increased confusion over the last two days. Increased sleepiness over two days (recent) is not conclusive of depression.The recent onset of urinary incontinence suggests that the delirium precipitant may be a urinary tract infection. Acute onset of increased confusion and fluctuating course, with disorganized thinking, inattention and altered level of consciousness are features of delirium, superimposed on the gradual deterioration due to dementia. The prevalence of delirium superimposed on dementia ranged from 22% to 89% of hospitalized and community populations aged 65 and older with dementia. Adverse events are associated with delirium in persons with dementia, including accelerated and long-term cognitive and functional decline, need for institutionalization, re-hospitalization, and increased mortality.The hallmark signs of normal pressure hydrocephalus (NPH) are dementia, gait disturbance, and urinary incontinence. Normal pressure hydrocephalus can be idiopathic or related to prior meningitis or subarachnoid hemorrhage. Ataxia is an important clinical sign of NPH.

Which statement regarding diffuse axonal injury is most accurate? A.It is rare in the pediatric population. B.Associated loss of consciousness is uncommon. C.Most commonly involves the watershed areas of the cerebral cortex. D.Initial neuroimaging studies generally lack specific findings.

D.Initial neuroimaging studies generally lack specific findings.82.85% Diffuse axonal injury can only be conclusively diagnosed microscopically, and the initial computerized tomography and magnetic resonance imaging scans generally have no specific findings. Axons closer to the surface of the brain are most susceptible and are seen at gray-white matter junctions. With more severe injuries, deeper structures, such as the midbrain, pons, corpus callosum, and the cerebellum may be damaged. DAI may occur in children and adults. DAI is the mechanism associated with the initial loss of consciousness after brain trauma.

A 48-year-old woman presents with a 2-month history of left thigh pain and left leg weakness with intermittent buckling at the knee. She denies any specific area of numbness. Left patellar reflex is diminished compared to the right. Left knee extension strength is 4/5; otherwise strength in the legs is 5/5. Which electrodiagnostic test is most likely to help with localization of the lesion? A.Motor nerve conduction studies B.Sensory nerve conduction studies C.F-wave and H-reflex studies D.Needle electromyography

D.Needle electromyography82.98% The clinical picture is suggestive of either a left femoral nerve lesion, lumbar radiculopathy, or lumbosacral plexopathy. In this type of situation, the needle examination is most helpful in localizing the lesion. Nerve conduction studies may be normal or nonspecific.

Which of the following best describes a typical symptom of lumbar spinal stenosis? A.Discogenic pain B.Distal sensory loss C.Urinary incontinence D.Neurogenic claudication

D.Neurogenic claudication93.85% Neurogenic claudication can present as unilateral or bilateral buttock, thigh, and/or lower leg pain or weakness. It is typically precipitated by walking and prolonged standing. The symptoms are generally relieved by flexion at the waist, such as leaning on a grocery cart when shopping. The forward-flexed posture is thought to dynamically widen the central canal thus de-creasing the mechanical compression of the spinal nerve roots.

A 47-year-old injured worker with low back pain is near maximum medical improvement (MMI) and is referred for a functional capacity evaluation (FCE). What knowledge can be obtained from FCEs for injured workers with chronic low back pain? A.Results only apply to occupational tasks, not to recreational activities B.FCEs are synonymous with work-hardening programs C.45% of patients experience recurrent back-related events within 1 year D.Performance on the floor-to-waist lift are as predictive as the number of failed tasks in the entire FCE protocol

D.Performance on the floor-to-waist lift are as predictive as the number of failed tasks in the entire FCE protocol Performance on the floor-to-waist lift are as predictive as the number of failed tasks in the entire FCE protocol. Limitations noted in an FCE apply to occupational and non-occupational (eg, recreational) activities. FCEs are used to estimate a patient's functional abilities, whereas work-hardening programs are designed to increase a patient's functional abilities. Following an FCE, 20% of patients experience back-related events within 1 year.

A patient with left hemiparesis left hemiparesis following a stroke, complains of left shoulder pain with ambulation. Which of the following is the most probable cause? A.Cervical radiculopathy B.Impingement syndrome C.Adhesive capsulitis D.Shoulder subluxation

D.Shoulder subluxation 89.97% Inferior subluxation of the glenohumeral joint occurs frequently following stroke. Pain in the shoulder is often felt in the upright position, since gravity further aggravates the subluxation. Shoulder pain after stroke is often attributed to shoulder subluxation; however, a causal relationship between subluxation and pain remains controversial. Shoulder subluxation has been associated complex regional pain syndrome type I, axillary neuropathy, and rotator cuff tears. Neuromuscular electrical stimulation to the deltoid and supraspinatus muscles can reduce subluxation and may reduce pain. Cervical radiculopathy is associated with radiating arm pain exacerbated by head movement and has a positive Spurling's sign. Impingement syndrome is made worse with overhead shoulder movement, particularly with Neer's maneuver. Adhesive capsulitis also is usually made worse with forceful range or attempts at shoulder level activity, but not usually ambulation.


Conjuntos de estudio relacionados

Prep-U Chapter 50: Assessment and management of patients with biliary disorders, PrepU Chapter 50: Biliary Disorders, PANCREATIC REVIEW

View Set

BADM Principles of Marketing: Unit 10

View Set

Varcarolis: Chapter 27 - Anger, Aggression, and Violence

View Set

7th Grade Civics - 3 Branches of Government

View Set

The Essentials of conflict Unit 1 Milestone

View Set

APCSP CH 15 internet study guide

View Set